7
$\begingroup$

A few days ago, I happened to go through the chapters on Radiation, and Photometry, studying them at quite an elementary level. I studied Wien's displacement law, and the dependence of luminous flux of a radiant source on the total radiant flux of the source, and the wavelength distribution. From studying them, I think that on changing the temperature of a radiant source, the relative-luminosities of the different component wavelengths of the radiation must change- for the following reason: on changing the temperature of the source, energy-density of the radiation redistributes itself over the entire range of the component wavelengths.

Now, I ask another (I cannot decide whether it is related with the previous one, or not) question. I assume a space of homogeneous R.I., and let, it has a remarkably high coefficient of viscosity. Let, a meteor passes through this space at a very, very high speed, away from an observer such that the viscous-drag is able to stop the meteor at a very very large distance from the observer, at rest, at origin (considering only two dimensional Cartesian coordinate system). Due to Doppler effect, the color-shift of the radiation (emitted due to action of viscous-drag on meteor-consequent heating-consequent temperature rise) should be towards red, but, due to Wien's displacement law, the dominance of the wavelength involved in the radiation (gradual increase in relative luminosity, in direction of violet-region---this is where I guess there might be the relation I spoke of earlier), gradually builds up in the direction of the violet-region.

My Question Is it possible that during the course of the journey, the color of radiation emitted by the meteor, as noticed by the observer, is white, due to wavelength compensation by Wien's displacement law, and Doppler effect?

Please, answer this question in a way as you seem most suitable, for a high-school student, with not much knowledge on Quantum-Mechanics (I guess, as I don't know what else I should know).

$\endgroup$
2
  • $\begingroup$ Are you essentially asking if there is a rate of change of temperature that could offset the observed Doppler effect? $\endgroup$
    – user6972
    Commented Jun 23, 2014 at 8:09
  • $\begingroup$ Why is the temperature of the meteor increasing in your example? Friction? $\endgroup$
    – garyp
    Commented Jun 26, 2014 at 3:36

1 Answer 1

2
$\begingroup$

Hmph, this is to do with the black body radiation spectrum.

Forgive me if I'm wrong, but from what I understand, the meteor is travelling away from the observer, but it is slowed down.

The image observed is redshifted, but then upon slowing down it begins to shift towards violet.

I initially thought that it would be a simple shift in hue; thus avoiding white. I also don't get how Wien's displacement law would apply (unless the meteor is hot enough to glow -around 1,000 Kelvin - due to particles causing friction) in a situation where the meteor is moving.

My answer: if Wien's displacement law applies to this, yes. Colours close to white exist within the black body radiation spectrum. This is also true (even with redshift; if the redshift gradually "fades", then it is possible to have a white-ish colour, but only when the wavelength compensation made by Wien's law is greater - therefore the object must be hotter).

See this and the image below:

http://phet.colorado.edu/sims/blackbody-spectrum/blackbody-spectrum_en.html

The simulation (use the above link) shows the relative "intensity values" of the three "primary" colours of light. According to the simulation, the "intensity values" of the three colours are almost equal between 5,000 and 6,000 Kelvin, producing the colour white.

Now, the addition of redshift would increase the "intensity value" of red. Therefore, in order to return it to a balance between the three colours, the object must be hotter.

https://upload.wikimedia.org/wikipedia/commons/b/ba/PlanckianLocus.png

If I am wrong or haven't explained it well enough, just comment and tell me why. I haven't come across Wien's displacement law before; it should have been made clearer in the question title.

Thanks.

$\endgroup$

Your Answer

By clicking “Post Your Answer”, you agree to our terms of service and acknowledge you have read our privacy policy.

Not the answer you're looking for? Browse other questions tagged or ask your own question.